From 6d447e25860f3cfcd22dc6097ac8c9ae79a4c8b2 Mon Sep 17 00:00:00 2001 From: wieerwill Date: Mon, 28 Mar 2022 10:08:47 +0200 Subject: [PATCH] kleine Korrekturen --- ...ogikprogrammierung - Prüfungsvorbereitung.pdf | 4 ++-- ...ogikprogrammierung - Prüfungsvorbereitung.tex | 16 ++++++++++++---- 2 files changed, 14 insertions(+), 6 deletions(-) diff --git a/Logik und Logikprogrammierung - Prüfungsvorbereitung.pdf b/Logik und Logikprogrammierung - Prüfungsvorbereitung.pdf index 97e2458..9d7010e 100644 --- a/Logik und Logikprogrammierung - Prüfungsvorbereitung.pdf +++ b/Logik und Logikprogrammierung - Prüfungsvorbereitung.pdf @@ -1,3 +1,3 @@ version https://git-lfs.github.com/spec/v1 -oid sha256:40fdddc14e91deff8deff055c23bd4bbd4b9c4df3d251a8575e862f748e2c77e -size 281978 +oid sha256:aa24931a4743249b0c827cd4f04048887bffff05fdb36040ab495671b6deb410 +size 283552 diff --git a/Logik und Logikprogrammierung - Prüfungsvorbereitung.tex b/Logik und Logikprogrammierung - Prüfungsvorbereitung.tex index e125742..07adb97 100644 --- a/Logik und Logikprogrammierung - Prüfungsvorbereitung.tex +++ b/Logik und Logikprogrammierung - Prüfungsvorbereitung.tex @@ -136,7 +136,7 @@ \begin{parts} \part Werte die Formel $\varpi_a=\lnot p \wedge \lnot\lnot p$ im Heytingschen Wahrheitswertebereich $H_{\mathbb{R}}$ aus für die $H_{\mathbb{R}}$-Belegung $B$ mit $B(p)=\mathbb{R}\backslash \{0\}$ \begin{solution} - $B_{H_{mathbb{R}}}(\lnot p \wedge \lnot\lnot p)= Inneres(\mathbb{R}/ p)\cap p= 1$ + $B_{H_{mathbb{R}}}(\lnot p \wedge \lnot\lnot p)= Inneres(\mathbb{R}/ p)\cap p= Inneres(\mathbb{R}\backslash\{\mathbb{R}\backslash\{0\}\})\cap \mathbb{R}\backslash\{0\}=\{0\}\cap \mathbb{R}\backslash\{0\} = \varnothing$ \end{solution} \part Überprüfe ob die Formel $\varphi_B=(\lnot p\rightarrow \lnot p)\rightarrow p$ eine $K_3$-Tautologie ist. Ist $\varphi_b$ eine $B_{\mathbb{R}}$ Tautologie? @@ -148,6 +148,10 @@ $\frac{1}{2}$ & $\frac{1}{2}$ & 1 & $\frac{1}{2}$ \\ 1 & 0 & 1 & 1 \\ \end{tabular} + + Keine $K_3$ Tautologie. + + Da keine $B$ Tautologie $\rightarrow$ keine $B_R$ Tautologie \end{solution} \part Überprüfe ob die semantische Folgeung $\{p\rightarrow q, q\rightarrow r\}\Vdash_B r\rightarrow\lnot p$ gilt. @@ -216,7 +220,7 @@ \item für 4.: $M_3=\{r,p\}$ \item für 1.: $M_4=\varnothing$ \end{enumerate} - \item $M_4=\varnothing \Rightarrow \varphi_b$ unerfüllbar + \item $M_4=\varnothing \Rightarrow \{\lnot\varphi_b\}$ unerfüllbar $\rightarrow \varphi$ Tautologie \end{itemize} \end{solution} \end{parts} @@ -290,9 +294,13 @@ \part Gebe die Regeln $(\forall-I)$, $(\exists-E)$ und $(GfG)$ inklusive Bedingung an \begin{solution} - $\varphi[x:=t]:\frac{\forall x\varphi}{\varphi[x:=t]}$ Bedingung: über keine Variable aus $t$ wird in $\varphi$ quantifiziert + $\forall-I:\frac{\varphi}{\forall x\varphi}$ Bedingung: x nicht frei in Hypothesen - $\exists x\varphi:\frac{\varphi[x:=t]}{\exists x\varphi}$ Bedingung: über keine Variable in $t$ wird in $\varphi$ quantifiziert + $\forall-E:\frac{\forall x\varphi}{\varphi[x:=t]}$ Bedingung: über keine Variable aus $t$ wird in $\varphi$ quantifiziert + + $\exists-I:\frac{\varphi[x:=t]}{\exists x\varphi}$ Bedingung: über keine Variable in $t$ wird in $\varphi$ quantifiziert + + $\exists-E:\frac{\exists x\varphi\quad \sigma}{\sigma}$ Bedingung: x weder frei in Hypothesen noch in $\sigma$ $(GfG): \frac{\varphi[x:=s]\quad s=t}{\varphi[x:=t]}$ Bedingung: über keine Variable aus $s$ oder $t$ wird in $\varphi$ quantifiziert \end{solution}